will mark bianleast
A point's coordinates are changed from (–7, –2) to (7, –2). What impact does this have on the location of the point?

The location of the point remains the same.

It is reflected over the y-axis.

The location of the point is shifted to the left.

It is reflected over the x-axis.

Answers

Answer 1

Answer: The answer would be "It is reflected over the x-axis"

Step-by-step explanation: The location wouldn't remain the same because the 7 is now a positive #. The y-axis hasn't changed. It's still the same -2. The location of the point would move, but it wouldn't move the the left. It would move to the right, because the negative numbers on the x-axis are on the left hand side, while the positive numbers are on the right hand side

Answer 2

Answer:When you reflect a point across the x-axis, the x-coordinate remains the same, but the y-coordinate is transformed into its opposite (its sign is changed). If you forget the rules for reflections when graphing, simply fold your paper along the x-axis (the line of reflection) to see where the new figure will be located.

Step-by-step explanation:


Related Questions

Arrange from smallest to largest
0.01, 1.01, 10.01, 1.001

Answers

Answer:

0.01 --> 1.001 ---> 1.01 ---> 10.01

The numbers in the ascending order:

0.01, 1.01, 1.001, 10.01.

What is ascending order?

Putting numbers in ascending order simply means to do it from smallest to largest.

Given:

0.01, 1.01, 10.01, 1.001.

The numbers from smallest to largest:

That means, in the ascending order.

So,

0.01, 1.01, 1.001, 10.01.

Therefore, 0.01, 1.01, 1.001, 10.01 are in the required order.

To learn more about the ascending order;

https://brainly.com/question/20681445

#SPJ2

Soledad has a storage box. The box is 6 1/2 inches long, 4 3/4 inches wide and 7 inches tall. She wants to run a border around the top of the box. How much border does she need?

Answers

Answer:

She need 10.5 inches border

Step-by-step explanation:

Length of box =[tex]\frac{1}{2} inches[/tex]

Breadth of box =[tex]4\frac{3}{4} inches[/tex]

Breadth of box =[tex]\frac{19}{4} inches[/tex]

Height of box = 7 inches

Now we are given that Soledad wants to run a border around the top of the box.

So, Length of top = Perimeter = [tex]2(l+b)=2(\frac{1}{2}+\frac{19}{4})=10.5[/tex]

Hence She need 10.5 inches border

Helppppp meeeee answer thissssssss

Answers

Answer:

(-1.025)^3 x (-1.025)^2=(- 1.025)^5

(-y)^3 x (-y)^2 x (-y)= y^6

5= odd, - stays

6= even, - becomes +

Answer:

5) (-1.025)^3* (-1.025)^2 = (-1.025)^5 = (-41/40)^5

10) -y^3 * -y^2 * -y

Multiplying an odd number of negative terms makes the product negative

= - y^3 * y^2 * y

= - y^6

What is -6 divided by -1.2?

Answers

Answer:

5.

Step-by-step explanation:

This is the same as saying:

-6/(-6/5).

To solve the equation, we multiply by the reciprocal and simplify. In this case, we multiply both sides of the equation by -5/6.

-6 * (-5/6) = 5.

The answer must be positive, as two negatives divided equal a positive.

I hope this helps!

The solution of expression after divide is,

⇒ 5

We have to given that;

⇒ - 6 is divided by - 1.2

Now, We can simplify as,

⇒ - 6 is divided by - 1.2

It can be written as,

⇒ - 6 ÷ - 1.2

⇒ - 6 / - 1.2

⇒ 6 / 1.2

⇒ 60 / 12

⇒ 5

Thus, The solution of expression after divide is,

⇒ 5

Learn more about the divide visit:

https://brainly.com/question/28119824

#SPJ6

Which function is represented by the graph below?

Answers

Answer:

y= 2x-4

Step-by-step explanation:

The y-intercept is at -4, it is a positive slope, plotting from the y-axis rise over run is equal to 2/1 which means that the slope is 2x. giving you y= 2x-4

The equation which satisfy to the graph is Y= 2x- 4

What is an equation?

An equation is a mathematical expression that contains an equals symbol. Equations often contain algebra.

According to graph

There are multiples points given on which line was passing

But we will select only which is clear

1st point = (0, 2)

2nd point = (-4,0 )

Now we will put values on equations and check which will satisfy

For 1st equation

Y= 2x- 4

Checking with 1st point

Putting x= 2

Y= 0 which satisfy

Checking for 2nd point

Putting x=0

Y = -4 satisfy

Hence, equation which satisfy to graph is Y= 2x- 4

To know more about equation here :

https://brainly.com/question/24169758

# SPJ2

HELP ASAP it says my questions should be at least 20 characters long so what I’m saying right now doesn’t mean anything.

Answers

Answer: See pic above for answer. I got it from Photomath

Plz put brainliest

14. Roger is on a playground swing, and he is swinging back and forth in such a way that the height, h, in feet, of the swing off the ground is given by the equation h=3cos(3π/2t) +5, where t is in seconds. How many seconds elapses between two consecutive times that the swing is at its maximum height?​

Answers

Answer:

The time [tex]t = \frac{3}{2}[/tex]  seconds elapses between two consecutive times that the swing is at its maximum height  'h' = 2

 Step-by-step explanation:

Explanation:-

Step(i):-

Given function [tex]h(t) = 3 cos (\frac{3\pi }{2 t} ) +5[/tex] ....(i)

By using derivative formulas

[tex]\frac{d cosx }{d x} = -sinx[/tex]

[tex]\frac{d x^{n} }{d x} = n x^{n-1}[/tex]

[tex]\frac{d t^{-1} }{d x} = -1 t^{-1-1} = - t^{-2} = \frac{-1}{t^{2} }[/tex]

Step(ii):-

Differentiating equation(i) with respective to 't'

[tex]h^{l} (t) = 3(-sin(\frac{3\pi }{2t})\frac{d}{dt} (\frac{3\pi }{2t } )+0[/tex]  ...(ii)

[tex]h^{l} (t) = 3(-sin(\frac{3\pi }{2t})(\frac{-3\pi }{2t^{2} } )+0[/tex]

Equating zero

[tex]h^{l} (t) = 3(-sin(\frac{3\pi }{2t})(\frac{-3\pi }{2t^{2} } )=0[/tex]

[tex]3(-sin(\frac{3\pi }{2t})(\frac{-3\pi }{2t^{2} } ) = 0[/tex]

on simplification , we get

[tex](sin(\frac{3\pi }{2t}) = 0[/tex]

now we use formulas

sin 0 = 0 and sinπ = 0

General solution

[tex](sin(\frac{3\pi }{2t}) = sin\pi[/tex]

[tex](\frac{3\pi }{2t}) = \pi[/tex]

Cancellation 'π' on both sides, we get

[tex]3 = 2 t[/tex]

Dividing '2' on both sides , we get

[tex]t = \frac{3}{2}[/tex]

Again differentiating with respective to 't' , we get  

[tex]h^{ll} (t) = 3(-cos(\frac{3\pi }{2t})(\frac{-3\pi }{2t^{2} } )+ (-3)(-sin(\frac{3\pi }{2t} )(\frac{6\pi }{2t^{3} }[/tex]

Put t= 3/2 and simplification

[tex]h^{ll} (t) < 0[/tex]

The maximum height

                   [tex]h(t) = 3 cos (\frac{3\pi }{2 t} ) +5[/tex]

               [tex]h(\frac{3}{2} ) = 3 cos (\frac{3\pi }{2(\frac{3}{2} )} )+5[/tex]

              [tex]h(\frac{3}{2} ) = 3 cos (\pi )+5 = -3+5 =2[/tex]

[tex]t = \frac{3}{2}[/tex]  seconds elapses between two consecutive times that the swing is at its maximum height  'h' = 2

Conclusion:-

The time [tex]t = \frac{3}{2}[/tex]  seconds elapses between two consecutive times that the swing is at its maximum height  'h' = 2

 

 

If f(x) = 2x + 1 and g(x) = x-2 what is the value of f(g(f(3)))? This is so confusing plz help me will mark brainliest A) 1 B) 3 C) 5 D) 7 E) 11

Answers

Answer:

11

Step-by-step explanation:

f(3) = 2*3+1 = 6+1 = 7

Then find g(7)

g(7) = 7-2 = 5

Then find f(5)

f(5) = 2*5 +1 = 10+1 = 11

f(g(f(3))) = 11

Answer:

11

Step-by-step explanation:

f(x)=2x + 1

f(3)= 2.3 + 1

f(3)=7

f(g(f(3))) = f(g(7))

Now, we have to find g(7)

g(x)=x -2

g(7)= 7 -2

g(7)=5

f(g(f(3))) = f(g(7)) = f(5)

now we have to find f(5)

f(x)=2x + 1

f(5)=2.5 + 1

f(5)=11

Hope this helps ^-^

Find the solution(s) to x^2- 14x + 49 = 0.
O A. x=-2 and x = 7
B. x= -1 and x = 14
C. x= 7 only
D. x = 7 and x = -7

Answers

Answer:

[tex] \boxed{C. \: x = 7 \: only} [/tex]

Step-by-step explanation:

[tex] = > {x}^{2} - 14x + 49 = 0 \\ \\ = > {x}^{2} - (7 + 7)x + 49 = 0 \\ \\ = > {x}^{2} - 7x - 7x + 49 = 0 \\ \\ = > x(x - 7) - 7(x - 7) = 0 \\ \\ = > (x - 7)(x - 7) = 0 \\ \\ = > {(x - 7)}^{2} = 0 \\ \\ = > x - 7 = 0 \\ \\ = > x = 7[/tex]

A. 256
B. 265
C. 297
D. 279​

Answers

Answer:

A.

Step-by-step explanation:

So calculate one of the triangles.

[tex]\frac{12*8}{2} =48[/tex]

48 * 4 = 192

192 + 64 = 256

I need help with is question ASAP please

Answers

The y-intercept looks like it's 0 and the slop looks like 6/2 or 3 or maybe 7/2



A new car is purchased for 23400 dollars. The value of the car depreciates at 11.5% per year. To the nearest year, how long will it be until the value of the car is 12700 dollars?

Answers

Answer: 5 years

Step-by-step explanation:

Using same formula as for Compound Interest:

12700 = 23400 (1-.115)^t

12700 = 23400 x 0.885^t

0.885^t = 127/234

Convert decimal into fraction:

177/200^t = 127/234

Take logarithm of both sides of thr equation:

t = log 177/200 (127/234)

t = 5.00242

00:00
Brice is finding the sum of 468 and 241 by breaking it into smaller problems
He uses place value and finds the sums of the hundreds, tens, and ones.
What is the sum of the tens? Enter your answer in the box.
1​

Answers

468+241=709 so 0 is in the tenth place because 6+4= 10 and one gets carried over to the 4+2

PLEASE HELP WILL GIVE BRAINLIEST AND 20 POINTS

solve this system of equations

3y-5x=12
y=1/3x

Answers

Answer:

x=-3

y=-1

(-3,-1)

Answer:

(-3, -1).

Step-by-step explanation:

I have attached the work to your problem.

Please see the attachment below.

I hope this helps!

The area of the base of the cone is 64π square millimeters, and the area of the lateral surface is 112π square millimeters. Find the radius r and slant height ℓ of the cone.

Answers

Answer:

Radius = 8 mm

Slant Height = 14 mm

Step-by-step explanation:

Base of a cone is a circle.

Area of circle is given as [tex]A=\pi r^{2}[/tex]

Where [tex]r[/tex] is the radius.

Given that

[tex]A = 64 \pi\ mm^2\\\Rightarrow \pi r^{2} = 64\pi \\\Rightarrow r^{2} = 64\\\Rightarrow r = 8\ mm[/tex]

Hence, radius is 8 mm.

Formula for Lateral Surface Area of a cone:

[tex]LSA = \pi rl[/tex]

Where [tex]r[/tex] is the radius and

[tex]l[/tex] is the slant height of cone

Given that [tex]LSA = 112 \pi\ mm^2[/tex]

[tex]\Rightarrow 112\pi = \pi \times 8 \times l\\\Rightarrow 8 \times l = 112\\\Rightarrow l = 14\ mm[/tex]

Hence, slant height is 14 mm

What is the equation in slope-intercept form of a line with slope of 5 and y-intercept of 3?

Answers

y=5x+3

Because the starting value(y intercept is 3) and the slope(x) is 5

BEST ANSWER GETS TO CHOOSE BRAINLESST OR FOLLOW!​

Answers

Answer:13 is>

14 is =

15 is >

16 is<

17 is<

18 is =

Step-by-step explanation:

I just know plz mark brainliest


[tex]( {2}^{ - 1} + {3}^{ - 1} )^{2} [/tex]
solve.

will give the brainliest​

Answers

Answer:

25/36

Step-by-step explanation:

(2^-1 + 3^-1)^2

(1/2 + 1/3)^2

(5/6)^2 = 25/36

25/36 or .694

2^-1= 1/2
3^-1=1/3

Add the together
5/6 square it and you get 25/36

Becky made 4 quarts of chicken noodle soup in a big pot, then served 112-cup bowls of soup to 6 people. How much soup is left in the pot?

Answers

Answer:

1qt 3c

Step-by-step explanation:

Is the relation a function?
{(-6, -1), (5,-1), (0, -1), (-2, -1), (3, -1)}

Answers

Answer:

yes it is a function

Step-by-step explanation:

For that no two x values are the same with different y values

evaluate the following expression -7x(7+9)

Answers

Answer:

-112

Step-by-step explanation:

Add

-7(7+9)

Multiply

-7x16

Wallah! You have the answer

-112

[tex]\text{Simplify the expression:}\\\\-7x(7+9)\\\\\text{Use the distributive property}\\\\-49x-63x\\\\\text{Combine like terms}\\\\\boxed{-112x}[/tex]

plz help i will give brainliest!!! 2.5(4x+10)−2(3x−15)

Answers

Answer:

4x+55

Step-by-step explanation:

Expand and simplify

10x+25x-6x+30

4x+55

Answer:

4x+55

Step-by-step explanation:

I used the PEDMAS to simplify the equation doing the parentheses first then addition and subtraction. The picture attached showed what order I did it in and I hope this really helped! Good LUCK on the rest of your homework.

soooo i need help can yall find the answer for me I put mad points so better get it right .

Answers

Answer:

1.08 * 10 ^8

Step-by-step explanation:

2.4 * 10^4   * 4.5 * 10^3

Multiply the constants

10.8

Add the exponents

10^(4+3) = 10^7

Put together

10.8 * 10 ^7

Put we can only have one number in front of the decimal

Move the decimal one place to the left and add one to the exponent

1.08 * 10 ^8

Which equation represents a line parallel to the line whose equations is -2x + 3y =
-4 and passes through the point (1,3)?

Answers

Answer:

2.  y - 3 = 2/3 (x - 1).

Step-by-step explanation:

-2x + 3y = -4

3y = 2x - 4

y = 2/3 x - 4/3 - so the slope is  2/3.

The slope of a line parallel to it is also 2/3.

It also passes through the point (1, 3).

Using the point-slope form of  a line:

y - y1 = m(x - x1) where m = the slope and (x1, y1) is a point on the line, we have:

y - 3 = 2/3 (x - 1)  <--- is the required equation.

Answer:

2

Step-by-step explanation:

parallel lines have same slope with different intercept

y= mx+b

m is going to be same with different b

so the given function is:

-2x+3y=-43y= 2x-4y= 2/3x - 4/3

Given options:

1. y-3= - 2/3(x-1)  ⇒ y= -2/3x +3 + 2/3 ⇒ y= -2/3x +11/3

it has different slope, so is not parallel

2. y-3= 2/3(x-1) ⇒ y= 2/3x+3-2/3 ⇒ y= 2/3x + 7/3

it has same slope, so is parallelit should be passing through point (1,3)3= 2/3+7/3 ⇒ 3=3, yes it does

3. y-3= -2/3(x+1) ⇒ y= - 2/3x +3- 2/3 ⇒ y= -2/3x + 7/3

it has different slope, is not parallel

4. y-3= 2/3(x+1) ⇒ y= 2/3x +3+ 2/3 ⇒ y= 2/3x +11/3

it has same slope, so is parallelit should be passing through point (1,3)3= 2/3+11/3 ⇒ 3≠13/3, no it doesn't

Tiffany cells 2 kinds of homemade tomato sauce

Answers

Answer:

b

Step-by-step explanation:

Let x represent the number of quarts of Tuscan sauce and

y represents the number of quarts of marinara sauce Tiffany makes.

A quart of Tuscan sauce requires 6 tomatoes and 1 cup of oil

x quarts requires 6x tomatoes and 1x cups of oil

A quart of her marinara sauce requires 5 tomatoes and 1.25 cups of oil

y quarts requires 5y tomatoes and 1.25 y cups of oil

She has 45 tomatoes and 10 cups of oil on hand.

So the constraints are

6x+5y≤45

1x+1.25y≤10

x>=0 and y>=0

After drawing a diagram of what's required to make 1 quart of Tuscan and 1 quart of Marinara sauce, you can more clearly choose answer choice (B) 6x + 5y < (or equal to) 45, x + 5/4y < (or equal to) 10, x>(or equal to)0, y>(or equal to)0.

please mark brainliest :)

the answer is B !!! hope it’s correct

Line I and h intersect at what point

Answers

Answer:

please show where the lines are graphed in a picture

Step-by-step explanation:

What is the midpoint of the segment below? (2,3)(-3,-2)

Answers

Answer:

(-0.5, 0.5)

Step-by-step explanation:

If those two points are endpoints, just use the midpoint formula.

(-0.5, 0.5)

Answer:

(-1/2,1/2)

Step-by-step explanation:

To find the midpoint, add the x coordinates and divide by 2

(2+-3)/2 = -1/2

add the y coordinates and divide by 2

(3+-2)/2 = 1/2

The midpoint is (-1/2,1/2)

please help as soon as possible
I WILL MARK YOU AS BRAINLIEST ​

Answers

Answer:

Last choice

Step-by-step explanation: Subtract 11 and then divide by 3. A would be greater than -2 and last choice shows that

If g(x) = -4x + 5, find g(2x-1).

Answers

Answer:

- 8x + 9

Step-by-step explanation:

To evaluate g(2x - 1) substitute x = 2x - 1 into g(x), that is

g(2x - 1)

= - 4(2x - 1) + 5 ← distribute parenthesis and simplify

= - 8x + 4 + 5

= - 8x + 9

Answer:

= - 8x + 9

Step-by-step explanation:

= - 8x + 4 + 5

= - 8x + 9

Solve:
-3(7p + 5) = 27
Helpppp

Answers

Answer:

-2

Step-by-step explanation:

-21p - 15 = 27

-21p = 42

p = -2

Answer:

-2

Step-by-step explanation:

-21p - 15 = 27

-21p = 42

p = -2

Other Questions
Solve for x. x/7=-3 -21-10421 5Select the correct answer.Which financial term refers to a reduction in the price of a product in order to obtain a response from consumers?loanOB.discountOC.interestODinvestment Im just curious. What does a person with no eyes experience. They cant see darkness because they have no eyes or can they? Please answer correctly !!!!!!!! Will mark brainliest !!!!!!!!!!!!! 9. Which is correct?Which famous Virginian said, "Give me liberty or give me death"?Which famous Virginian said, "Give me liberty or give me death?"Which famous Virginian said "Give me liberty or give me death"?Which famous Virginian said "Give me liberty or give me death?" Which of these best describes the purpose for "hands-only" CPR?AHands-only CPR is better and more effective than full CPR and is now the best way to provide CPR for any patient by any responderBHands-only CPR reduces risk of liability and increases oxygenation better than CPR with mouth-to-mouth resuscitationCHands-only CPR is now the only method for providing CPR regardless your level of education and expertiseDHands-only CPR was designed for those who are not trained or feel uncomfortable delivering mouth-to-mouth breaths. It doesn't replace traditional CPR. If you need to park your vehicle beside a highway,Choose an answer:A. You must move completely off the hardsurface of the highway unless parkingspaces are marked on the hard surface.B. You may park on the hard surface of thehighway during daylight hours, but not atnight.C. You can park on the hard surface of thehighway if there are no signs prohibitingparking. The case of Furman versus Georgia resulted in a reinterpretation of the meaning of Question 1 (1 point)What is the solution set to the equation x+1=2Question 2 (1 point)Consider the equation x +1 = 2.Rewrite the equation by multiplying both sides by x +1DO NOT USE A MULTIPLICATION SYMBOLQuestion 3 (1 point)Solve the equation for5(2x - 4) - 11 = 4 + 3xplease answer all if possible. Thanks HELPP ASAP !! Examine the table of contents . Table of Contents Fear and its Effects 3. Physical EffectsMental Effects Emotional Effects Just Say " No ! " to Fear 35. Breathing Exercises Meditation Yoga Free from Fear 52. Living Free Seeking Support Staying Sane IndexIn which chapter would a reader find information on meditation? Fear and its Effects Free From Fear Just Say No to Fear Meditation Link is coloring a triforce, which consists of four equilateral triangles and is depicted below. He has three colors to use: gold, black, and green. So that it remains recognizable, he doesn't want to color any two triangles the same color if they share a side. How many different ways can he color the triforce? (Two colorings that differ by rotation are considered distinct.) Please answer correctly !!!!!!!! Will mark brainliest !!!!!!!!!! Evidence left behind by participants or observers is an example of what kind of resource? A) primary B) hearsay C) secondary D) hieroglyphics determine the kinetic energy of 745-kg roller coaster car that is moving with a speed of 14.9 m/s Please answer correctly !!!!!!! Will mark brainliest !!!!!!!!!!! Which of these choices are quadratic equations? Check all that apply.A. 2x 1 = 0B. 2x2 - 1 = 0C. 2x2 - 1D. 3x2 + 5x - 1 = 0E. x - x +5=0F. - 4x2 + x = 0 A moving walkway has a speed of 0.5 m/s to the east. A stationary observersees a man walking on the walkway with a velocity of 0.8 m/s to the east.What is the man's velocity relative to the moving walkway? Suppose tortilla chips cost 22.5 cents per ounce. If a bag costs $3.07, how many ounces are in the bag of chips? Round your answer to the nearest hundredth, if necessary. Perform algorithm time measurement for all three quadratic sorting algorithms for Best Case (already sorted), Average Case (randomly generated), and Worst Case (inverse sorted) inputs and compare them with O(N*logN) sorting algorithm using Arrays.sort() method. Use arrays sizes of 50K, 100K, and 200K to produce timing results. What is a jet stream?